Difference between revisions of "1973 USAMO Problems/Problem 5"

m (Problem)
Line 3: Line 3:
  
 
==Solution==
 
==Solution==
{{solution}}
+
 
 +
Let the three distinct prime number be <math>p</math>, <math>q</math>, and <math>r</math>
 +
 
 +
WLOG, let <math>p<q<r</math>
 +
 
 +
Assuming that the cube roots of three distinct prime numbers <math>can</math> be three terms of an arithmetic progression.
 +
 
 +
Then,
 +
 
 +
<cmath>q^{\dfrac{1}{3}}=p^{\dfrac{1}{3}}+md</cmath>
 +
 
 +
<cmath>r^{\dfrac{1}{3}}=p^{\dfrac{1}{3}}+nd</cmath>
 +
 
 +
where <math>m</math>, <math>n</math> are distinct integer, and d is the common difference in the progression (it's not necessary an integer)
 +
 
 +
<cmath>nq^{\dfrac{1}{3}}-mr^{\dfrac{1}{3}}=(n-m)p^{\dfrac{1}{3}}-----(1)</cmath>
 +
 
 +
<cmath>n^{3}q-3n^{2}mq^{\dfrac{2}{3}}r^{\dfrac{1}{3}}+3nm^{2}q^{\dfrac{1}{3}}r^{\dfrac{2}{3}}-m^{3}r=(n-m)^{3}p</cmath>
 +
 
 +
<cmath>3nm^{2}q^{\dfrac{1}{3}}r^{\dfrac{2}{3}}-3n^{2}mq^{\dfrac{2}{3}}r^{\dfrac{1}{3}}=(n-m)^{3}p+m^{3}r-n^{3}q</cmath>
 +
 
 +
<cmath>(3nmq^{\dfrac{1}{3}}r^{\dfrac{1}{3}})(mr^{\dfrac{1}{3}}-nq^{\dfrac{1}{3}})=(n-m)^{3}p+m^{3}r-n^{3}q-----(2)</cmath>
 +
 
 +
<cmath>nq^{\dfrac{1}{3}}-mr^{\dfrac{1}{3}}=(n-m)p^{\dfrac{1}{3}}-----(1)</cmath>
 +
 
 +
<cmath>mr^{\dfrac{1}{3}}-nq^{\dfrac{1}{3}}=(m-n)p^{\dfrac{1}{3}}</cmath>
 +
 
 +
<cmath>(3nmq^{\dfrac{1}{3}}r^{\dfrac{1}{3}})((m-n)p^{\dfrac{1}{3}})=(n-m)^{3}p+m^{3}r-n^{3}q-----(1) and (2)</cmath>
 +
 
 +
<cmath>q^{\dfrac{1}{3}}r^{\dfrac{1}{3}}p^{\dfrac{1}{3}}=\dfrac{(n-m)^{3}p+m^{3}r-n^{3}q)}{(3mn)(m-n)}</cmath>
 +
 
 +
<cmath>(pqr)^{\dfrac{1}{3}}=\dfrac{(n-m)^{3}p+m^{3}r-n^{3}q)}{(3mn)(m-n)}</cmath>
 +
 
 +
now using the fact that <math>p</math>, <math>q</math>, <math>r</math> are distinct primes, <math>pqr</math> is not a cubic
 +
 
 +
Thus, the LHS is irrational but the RHS is rational, which causes a contradiction
 +
 
 +
Thus, the cube roots of three distinct prime numbers cannot be three terms of an arithmetic progression.
  
 
{{USAMO box|year=1973|num-b=4|after=Last Problem}}
 
{{USAMO box|year=1973|num-b=4|after=Last Problem}}
  
 
[[Category:Olympiad Number Theory Problems]]
 
[[Category:Olympiad Number Theory Problems]]

Revision as of 19:39, 10 August 2009

Problem

Show that the cube roots of three distinct prime numbers cannot be three terms (not necessarily consecutive) of an arithmetic progression.

Solution

Let the three distinct prime number be $p$, $q$, and $r$

WLOG, let $p<q<r$

Assuming that the cube roots of three distinct prime numbers $can$ be three terms of an arithmetic progression.

Then,

\[q^{\dfrac{1}{3}}=p^{\dfrac{1}{3}}+md\]

\[r^{\dfrac{1}{3}}=p^{\dfrac{1}{3}}+nd\]

where $m$, $n$ are distinct integer, and d is the common difference in the progression (it's not necessary an integer)

\[nq^{\dfrac{1}{3}}-mr^{\dfrac{1}{3}}=(n-m)p^{\dfrac{1}{3}}-----(1)\]

\[n^{3}q-3n^{2}mq^{\dfrac{2}{3}}r^{\dfrac{1}{3}}+3nm^{2}q^{\dfrac{1}{3}}r^{\dfrac{2}{3}}-m^{3}r=(n-m)^{3}p\]

\[3nm^{2}q^{\dfrac{1}{3}}r^{\dfrac{2}{3}}-3n^{2}mq^{\dfrac{2}{3}}r^{\dfrac{1}{3}}=(n-m)^{3}p+m^{3}r-n^{3}q\]

\[(3nmq^{\dfrac{1}{3}}r^{\dfrac{1}{3}})(mr^{\dfrac{1}{3}}-nq^{\dfrac{1}{3}})=(n-m)^{3}p+m^{3}r-n^{3}q-----(2)\]

\[nq^{\dfrac{1}{3}}-mr^{\dfrac{1}{3}}=(n-m)p^{\dfrac{1}{3}}-----(1)\]

\[mr^{\dfrac{1}{3}}-nq^{\dfrac{1}{3}}=(m-n)p^{\dfrac{1}{3}}\]

\[(3nmq^{\dfrac{1}{3}}r^{\dfrac{1}{3}})((m-n)p^{\dfrac{1}{3}})=(n-m)^{3}p+m^{3}r-n^{3}q-----(1) and (2)\]

\[q^{\dfrac{1}{3}}r^{\dfrac{1}{3}}p^{\dfrac{1}{3}}=\dfrac{(n-m)^{3}p+m^{3}r-n^{3}q)}{(3mn)(m-n)}\]

\[(pqr)^{\dfrac{1}{3}}=\dfrac{(n-m)^{3}p+m^{3}r-n^{3}q)}{(3mn)(m-n)}\]

now using the fact that $p$, $q$, $r$ are distinct primes, $pqr$ is not a cubic

Thus, the LHS is irrational but the RHS is rational, which causes a contradiction

Thus, the cube roots of three distinct prime numbers cannot be three terms of an arithmetic progression.

1973 USAMO (ProblemsResources)
Preceded by
Problem 4
Followed by
Last Problem
1 2 3 4 5
All USAMO Problems and Solutions